TheOverlord

New Member
ارسال ها
159
لایک ها
282
امتیاز
0
پاسخ : ماراتن نظریه ی اعداد (سطح پیشرفته)

این راه درسته؟:17:

فرض کنید که:


حالا اگه
، n امین عدد اول باشه اون وقت
اما که تناقضه.
درست است؟ برای مثال اگر یک چندجمله ای را در نظر بگیرید این حد غلط میشود! در حقیقت نسبت این دو به یک میل میکند ولی اختلافشان ممکن است به بینهایت میل کند.
 

aras2213

New Member
ارسال ها
216
لایک ها
228
امتیاز
0
پاسخ : ماراتن نظریه ی اعداد (سطح پیشرفته)

درست است؟ برای مثال اگر یک چندجمله ای را در نظر بگیرید این حد غلط میشود! در حقیقت نسبت این دو به یک میل میکند ولی اختلافشان ممکن است به بینهایت میل کند.
اگه ثابت کنیم درجشون برابره درست میشه؟

فکر کنم این جوری بشه گفت که درجشون برابره.

که c یه عدد ثابته.حالا اگه درجشون برابر نباشه...................................................................................................فکر کنم تناقض بشه:4:
 

TheOverlord

New Member
ارسال ها
159
لایک ها
282
امتیاز
0
پاسخ : ماراتن نظریه ی اعداد (سطح پیشرفته)

اگه ثابت کنیم درجشون برابره درست میشه؟

فکر کنم این جوری بشه گفت که درجشون برابره.

که c یه عدد ثابته.حالا اگه درجشون برابر نباشه...................................................................................................فکر کنم تناقض بشه:4:
در حقیقت اگر ثابت کنیم درجه صورت از مخرج نابیشتر است درست میشود و نامساوی شما هم همین نتیجه را میدهد، اگر از حد استفاده کنید!
 

aras2213

New Member
ارسال ها
216
لایک ها
228
امتیاز
0
پاسخ : ماراتن نظریه ی اعداد (سطح پیشرفته)

سوال بعد:دنباله
این گونه تعریف شده است:


نشان دهید برای هر n طبیعی ،
مربع کامل است.
 

Dadgarnia

New Member
ارسال ها
1,350
لایک ها
1,127
امتیاز
0
پاسخ : ماراتن نظریه ی اعداد (سطح پیشرفته)

سوال بعد:دنباله
این گونه تعریف شده است:


نشان دهید برای هر n طبیعی ،
مربع کامل است.
به راحتي مي توانيم بدست بياوريم
حالا دنباله ي
را به شكل زير تعريف مي كنيم:

به طور مشابه مي توانيم بدست بياوريم
داريم:

و حكم ثابت مي شود.
 

aras2213

New Member
ارسال ها
216
لایک ها
228
امتیاز
0
پاسخ : ماراتن نظریه ی اعداد (سطح پیشرفته)

چون آقای Dadgarnia سوال نداشتن سوال بعد رو هم به درخواست ایشون من میذارم:

سوال بعد:همه اعداد طبیعی n را بیابید که
مربع کامل شود.
 

AHZolfaghari

Well-Known Member
ارسال ها
935
لایک ها
1,654
امتیاز
93
پاسخ : ماراتن نظریه ی اعداد (سطح پیشرفته)

چون آقای Dadgarnia سوال نداشتن سوال بعد رو هم به درخواست ایشون من میذارم:

سوال بعد:همه اعداد طبیعی n را بیابید که
مربع کامل شود.
دو طرف رو با 121 جمع میکنیم و از این که اگه p عدد 4k+3 باشه و جمع دو تا مربع کامل رو عاد کنه اونوقت هرکدومشون رو عاد میکنه استفاده میکنیم . و به تناقض می رسیم

---- دو نوشته به هم متصل شده است ----

سوال بعد :
تمام m,n های طبیعی را بیابید بطوریکه :
 

m-saghaei

New Member
ارسال ها
338
لایک ها
258
امتیاز
0
پاسخ : ماراتن نظریه ی اعداد (سطح پیشرفته)

دو طرف رو با 121 جمع میکنیم و از این که اگه p عدد 4k+3 باشه و جمع دو تا مربع کامل رو عاد کنه اونوقت هرکدومشون رو عاد میکنه استفاده میکنیم . و به تناقض می رسیم

---- دو نوشته به هم متصل شده است ----

سوال بعد :
تمام m,n های طبیعی را بیابید بطوریکه :
جوابهاش اینا میشه؟اگه میشه تا راهمو بزارم!
(1,2),(2,1),(1,3),(3,1),(2,2),(2,5),(5,2),(3,5),(5,3)

---- دو نوشته به هم متصل شده است ----

دیگه میزاریم هرچه باد آباد!!!!!!!!!
داریم
پس
و چون
تیجه میشه
و همینطور اینا رو میتونیم بدست بیاریم:
و
و ...
همینطور چون
پس
که نتیجه میشه
پس اگه m مساوی 1 نباشه داریم
که نتیجه میشه اگه m بزرگتر از 3 باشه اونوقت داریم

پس :
اگه m=1 اونوقت n=2,3
اگه m=2 اونوقت n=2,5
اگه m=3 اونوقت n=5
اگه m>3 اونوقت باید n=m و n=m+1 رو بررسی کنیم.
که در کل تمامی جواب ها میشن: (1,2),(2,1),(1,3),(3,1),(2,2),(2,5),(5,2),(3,5),(5,3)
 

AHZolfaghari

Well-Known Member
ارسال ها
935
لایک ها
1,654
امتیاز
93
پاسخ : ماراتن نظریه ی اعداد (سطح پیشرفته)

جوابهاش اینا میشه؟اگه میشه تا راهمو بزارم!
(1,2),(2,1),(1,3),(3,1),(2,2),(2,5),(5,2),(3,5),(5,3)

---- دو نوشته به هم متصل شده است ----

دیگه میزاریم هرچه باد آباد!!!!!!!!!
داریم
پس
و چون
تیجه میشه
و همینطور اینا رو میتونیم بدست بیاریم:
و
و ...
همینطور چون
پس
که نتیجه میشه
پس اگه m مساوی 1 نباشه داریم
که نتیجه میشه اگه m بزرگتر از 3 باشه اونوقت داریم

پس :
اگه m=1 اونوقت n=2,3
اگه m=2 اونوقت n=2,5
اگه m=3 اونوقت n=5
اگه m>3 اونوقت باید n=m و n=m+1 رو بررسی کنیم.
که در کل تمامی جواب ها میشن: (1,2),(2,1),(1,3),(3,1),(2,2),(2,5),(5,2),(3,5),(5,3)
بله درسته جواب ها همین ها بودند که فرمودید ! سوال 4 المپیاد جهانی 1994 بود !سوال بعدی رو لطف کنید شما بذارید
 

m-saghaei

New Member
ارسال ها
338
لایک ها
258
امتیاز
0
پاسخ : ماراتن نظریه ی اعداد (سطح پیشرفته)

سوال بعد:
همه ی زوج ها از عددهای طبیعی مانند (a,b) را طوری پیدا کنید که
 

Dadgarnia

New Member
ارسال ها
1,350
لایک ها
1,127
امتیاز
0
پاسخ : ماراتن نظریه ی اعداد (سطح پیشرفته)

سوال بعد:
همه ی زوج ها از عددهای طبیعی مانند (a,b) را طوری پیدا کنید که
داريم:

كه تناقضه مگر اين كه b يك باشه و a عددي زوج. سوال بعدي رو يكي از دوستان بذاره.
 

m-saghaei

New Member
ارسال ها
338
لایک ها
258
امتیاز
0
پاسخ : ماراتن نظریه ی اعداد (سطح پیشرفته)

داريم:

كه تناقضه مگر اين كه b يك باشه و a عددي زوج. سوال بعدي رو يكي از دوستان بذاره.
اگه تو صورت سوال به جای
- ,
- باشه چه جوری میشه؟یعنی این طوری:


آخه بین
و
شک دارم!!!
 

AHZolfaghari

Well-Known Member
ارسال ها
935
لایک ها
1,654
امتیاز
93
پاسخ : ماراتن نظریه ی اعداد (سطح پیشرفته)

اگه تو صورت سوال به جای
- ,
- باشه چه جوری میشه؟یعنی این طوری:


آخه بین
و
شک دارم!!!
b^3 هستش . سوال جهانی بوده . که بخوام خلاصه بگم باید عبارت رو برابر یه k قرار بدید . بعد یه معادله درجه دو برحسب a درست میکنیم و ازش دلتا میگیریم.
دلتا باید مربع کامل باشه . بعد دلتا رو بصورت جمع یه مربع کامل و یه عدد دیگه می نویسیم . (x^2 + a )
چون مربع کامله پس a بزرگ تر مساوی 2x+1 بعد یه نتایج خوبی میده ! حالا اگه حلش کردی خودش حلشو بذار !!!!!
 

math1998

New Member
ارسال ها
336
لایک ها
224
امتیاز
0
پاسخ : ماراتن نظریه ی اعداد (سطح پیشرفته)

همه ی
را پیدا کنید که عدد
موجود باشد که
 

Dadgarnia

New Member
ارسال ها
1,350
لایک ها
1,127
امتیاز
0
پاسخ : ماراتن نظریه ی اعداد (سطح پیشرفته)

همه ی
را پیدا کنید که عدد
موجود باشد که
اگه n فرد باشه با توجه به اینکه
عددی به شکل
هست
حتما مقسوم علیه اولی به شکل
و بزرگتر از 3 داره پس این عدد اول باید 9 رو بشماره که تناقضه. حالا اگه n به شکل
باشه که k عددی فرد و بزرگتر از 1 و
عددی طبیعی است داریم:

می دونیم که
عامل اولی به شکل
و بزرگتر از 3 داره پس مثل قبل می تونیم به تناقض برسیم. حالا فرض کنید n به شکل
باشه ثابت می کنیم چنین m ای وجود داره.
واضحه که m باید بر 3 بخشپذیر باشه پس فرض کنید
و p مقسوم علیه اولی (به جز 3) از
باشه پس ما باید ثابت کنیم t ای وجود داره که داشته باشیم (
توان p در
است):

پس کافیه t وارون ضربی
به پیمانه ی
باشه که می دونیم چنین t ای وجود داره. پس چیزی که می خواستیم ثابت شد.
 

m-saghaei

New Member
ارسال ها
338
لایک ها
258
امتیاز
0
پاسخ : ماراتن نظریه ی اعداد (سطح پیشرفته)

سوال بعدی:
اعداد صحیح
را چنان بیابید که
 

Dadgarnia

New Member
ارسال ها
1,350
لایک ها
1,127
امتیاز
0
پاسخ : ماراتن نظریه ی اعداد (سطح پیشرفته)

سوال بعدی:
اعداد صحیح
را چنان بیابید که
اگه يكي از
صفر باشه به راحتي مي تونيم ببينيم كه جوابي وجود نداره در غير اين صورت اگه دو طرف رو به پيمانه ٨ در نظر بگيريم بدست مياد
حالا دو طرف رو به پيمانه ي ٧ در نظر مي گيريم مي دونيم:

و
به پيمانه ي ٧ براير با يك يا ١- است پس
. داريم:

مثل قبل اگر دو طرف را به پيمانه ي ٧ در نظر بگيريم بدست مياد
و اگر به پيمانه ي ١٩ در نظر بگيريم بدست مياد
با جايگذاري اين روابط در رابطه ي بالا داريم:

كه به راحتي مي توان چك كرد برقرار نمي باشد پس جوابي وجود نداره.
 

m-saghaei

New Member
ارسال ها
338
لایک ها
258
امتیاز
0
پاسخ : ماراتن نظریه ی اعداد (سطح پیشرفته)

اگه يكي از
صفر باشه به راحتي مي تونيم ببينيم كه جوابي وجود نداره در غير اين صورت اگه دو طرف رو به پيمانه ٨ در نظر بگيريم بدست مياد
حالا دو طرف رو به پيمانه ي ٧ در نظر مي گيريم مي دونيم:

و
به پيمانه ي ٧ براير با يك يا ١- است پس
. داريم:

مثل قبل اگر دو طرف را به پيمانه ي ٧ در نظر بگيريم بدست مياد
و اگر به پيمانه ي ١٩ در نظر بگيريم بدست مياد
با جايگذاري اين روابط در رابطه ي بالا داريم:

كه به راحتي مي توان چك كرد برقرار نمي باشد پس جوابي وجود نداره.
قسمت اولشو یه اشتباه کوچولو کردید چون اگه
اونوقت
جواب میشه!
 

Dadgarnia

New Member
ارسال ها
1,350
لایک ها
1,127
امتیاز
0
پاسخ : ماراتن نظریه ی اعداد (سطح پیشرفته)

سوال بعد:
تمام اعداد طبيعي
را بيابيد به طوريكه
عددي طبيعي باشد.
 

aras2213

New Member
ارسال ها
216
لایک ها
228
امتیاز
0
پاسخ : ماراتن نظریه ی اعداد (سطح پیشرفته)

سوال بعد:
تمام اعداد طبيعي
را بيابيد به طوريكه
عددي طبيعي باشد.
اول برای n=1,2,3 دستی چک میکنیم.پس فرض کنید n از 3 بزرگتر باشه.نشون میدیم که
.

برای سمت راست نابرابری
. اما
.پس

نابرابری سمت راست درسته.


برای سمت چپ نابرابری
پس کافیه که ثابت کنیم
که این هم درسته.پس برای n های بزرگتر از 3 جواب نداریم.

---- دو نوشته به هم متصل شده است ----

سوال بعد:همه ی اعداد اول p,q را بیابید که
بر
بخش پذیر باشد.
 
بالا